AFP Questions 4

Ace your homework & exams now with Quizwiz!

Which one of the following hospitalized patients is the most appropriate candidate for thromboembolism prophylaxis with enoxaparin (Lovenox)? (check one) A. An ambulatory 22-year-old obese male admitted for an appendectomy B. A 48-year-old male with atrial fibrillation on chronic therapeutic anticoagulation, admitted for cellulitis C. A 48-year-old male with end-stage liver disease and coagulopathy D. A 52-year-old female on chronic estrogen therapy, admitted with severe thrombocytopenia E. A 67-year-old female with hemiparesis, admitted for community-acquired pneumonia

. A 67-year-old female with hemiparesis, admitted for community-acquired pneumonia Venous thromboembolism is a frequent cause of preventable death and illness in hospitalized patients. Prophylaxis is generally recommended for patients over the age of 40 who have limited mobility for 3 days or more and have at least one of the following risk factors: acute infectious disease, New York Heart Association class III or IV heart failure, acute myocardial infarction, acute respiratory disease, stroke, rheumatic disease, inflammatory bowel disease, previous venous thromboembolism, older age (especially >75 years), recent surgery or trauma, immobility or paresis, obesity (BMI >30 kg/m2), central venouscatheterization, inherited or acquired thrombophilic disorders, varicose veins, or estrogen therapy.

The U.S. Preventive Services Task Force (USPSTF) recommends screening for asymptomatic bacteriuria with a urine culture for pregnant women at ______ weeks gestation or at the first prenatal visit if it occurs later (A recommendation).

12-16 The USPSTF recommends against screening for asymptomatic bacteriuria in men and nonpregnant women (D recommendation). Screening for asymptomatic bacteriuria is not recommended for long-term care residents or patients with indwelling bladder catheters without symptoms of a UTI (catheter-associated asymptomatic bacteriuria).

According to Global Initiative for Obstructive Lung Disease (GOLD) guidelines, an inhaled corticosteroid should be added for frequent exacerbations and an FEV1 <______%.

50

In which one of the following scenarios would additional consent from a child's parent or guardian be necessary before proceeding with treatment in nearly every U.S. state? (check one) A. A 6-year-old female with divorced parents who lives primarily with her mother is brought to the clinic by her father to discuss his concerns of possible abuse B. An 8-year-old unconscious male is brought to the emergency department by a neighbor after falling out of a tree and striking his head C. A 13-year-old male is brought to the clinic by a babysitter with a note giving permission to treat signed by a parent D. A 15-year-old female who is considered emancipated under state law comes to your office to discuss family planning E. A 16-year-old female who has driven herself to her clinic appointment reports a 2-day history of ear pain; she says her mother made this appointment for her

A 16-year-old female who has driven herself to her clinic appointment reports a 2-day history of ear pain; she says her mother made this appointment for her Children below the age of majority must have proof of permission to treat from a parent or guardian for non-emergent care. This does not apply to emergency situations in which a delay in care could result in serious harm. Another exception to parental consent is when a child is considered emancipated under state law. This can happen with a court order, or (in some states) if the child is married, is a parent, is in the military, or is living independently. Either biologic parent can consent to treatment unless one of them is explicitly denied guardianship. If a child presents with a non-emergent condition and does not have evidence of permission from a parent or guardian, permission should be sought before the physician interaction takes place.

A 90-year-old female with severe dementia is seen in the emergency department for a left knee strain. She was at home alone for 2 hours and no fall or injury was witnessed. A radiograph of the knee is negative, and she is referred to you for follow-up the next day. At the follow-up visit the patient is confused and agitated, and cries out at any attempt to examine her. She is unable to bear weight on her left leg and it appears to be externally rotated. Which one of the following would be most appropriate at this point? (check one) A. Reassurance and pain medication B. A repeat radiograph of the left knee C. A radiograph of the left hip D. A radiograph of the lumbosacral spine E. MRI of the left knee

A fractured hip is possible and must be ruled out since there is difficulty bearing weight and the leg is externally rotated. Examination of a patient with severe dementia can be extremely difficult. Other findings with a fractured hip would include pain elicited on rotation and groin pain when applying an axial load. If the hip radiograph is negative, MRI of the knee may be considered.

A 36-year-old female sees you for a 6-week postpartum visit. Her pregnancy was complicated by gestational diabetes mellitus. Her BMI at this visit is 33.0 kg/m2 and she has a family history of diabetes.Which one of the following is this patient's greatest risk factor for developing type 2 diabetes in the future? (check one) A. Her age B. Obesity C. The history of a completed pregnancy D. The history of gestational diabetes E. The family history of diabetes

A history of gestational diabetes mellitus (GDM) is the greatest risk factor for future development of diabetes mellitus. It is thought that GDM unmasks an underlying propensity to diabetes. While a healthy pregnancy is a diabetogenic state, it is not thought to lead to future diabetes. This patient's age is not a risk factor. Obesity and family history are risk factors for the development of diabetes, but having GDM leads to a fourfold greater risk of developing diabetes, independent of other risk factors (SOR C). It is thought that 5%-10% of women who have GDM will be diagnosed with type 2 diabetes within 6 months of delivery. About 50% of women with a history of GDM will develop type 2 diabetes within 10 years of the affected pregnancy.

A 2-year-old female is brought to your office with an occasional barking cough that began late last night. The child has mild intercostal indrawing and no stridor at rest.Which one of the following would be most appropriate to help improve this child's symptoms? (check one) A. Humidification B. A helium-oxygen mixture (heliox) C. Oral dexamethasone D. Nebulized albuterol E. Nebulized budesonide (Pulmicort Respules)

A single dose of oral dexamethasone improves symptoms in children with mild croup when compared with placebo. It is as effective for reducing croup symptoms as nebulized budesonide and is less distressing for the child. There is currently no evidence from randomized, controlled trials to support the use of humidification or a helium-oxygen mixture to reduce the symptoms of croup.

What is recommended as prophylaxis in asymptomatic patients after a known tick bite.

A single-dose treatment with doxycycline or amoxicillin is recommended as prophylaxis in asymptomatic patients after a known tick bite. This is only recommended for tick attachment longer than 36 hours, or of unknown duration. For facial nerve palsy, treatment with doxycycline or amoxicillin for 14 days is effective. Patients with more severe neurologic manifestations of Lyme disease, such as altered mental status, meningoencephalitis, or other cranial nerve palsies, require longer courses of antibiotics, usually intravenously.

A 29-year-old mother of three young children asks your opinion on giving probiotics to her children. Which one of the following is a benefit of the use of probiotics in children? (check one) A. They reduce diarrhea associated with irritable bowl syndrome B. They reduce colic symptoms in formula-fed infants C. They prevent the development of allergies D. They prevent antibiotic-associated diarrhea

A wealth of evidence-based research has established the benefits of probiotics, especially in children. Probiotics, particularly Saccharomyces boulardii , have been shown to prevent the antibiotic-associated diarrhea that occurs in 5%-30% of children who receive antibiotics (SOR A). The number needed to treat to prevent one case of diarrhea is 10. Probiotics reduce the pain associated with irritable bowel syndrome (IBS) but have not been shown to be helpful in reducing diarrhea or constipation in pediatric IBS patients (SOR A).In breastfed infants, probiotics reduce daily crying time by up to an hour. Similar benefits have not been found in formula-fed infants or infants who are combining breastfeeding and formula. Probiotics have not been shown to prevent colic in any infants. Other benefits of probiotics include the prevention of eczema and upper respiratory infections (SOR A). However, their use has not been shown to prevent allergies and asthma.

A 54-year-old male presents to your office with a 2-day history of mild right anterior chest pain with deep breathing. He reports that it has been sharp and constant and is preventing him from sleeping. He also describes shortness of breath and a cough productive of white sputum. A chest radiograph is normal. An EKG reveals right bundle branch block. Which one of the following would you order next? (check one) A. A D-dimer level B. Compression ultrasonography C. Echocardiography D. A ventilation-perfusion scan of the lungs E. CT angiography of the lungs

A. A D-dimer level The PIOPED study reported the following incidence of common symptoms of pulmonary embolism : Dyspnea (73%) Pleuritic chest pain (66%) Cough (37%) Validated clinical prediction rules can be used to estimate the pretest probability of deep vein thrombosis (DVT) and pulmonary embolism in a patient with dyspnea and chest pain, and to guide further evaluation (SOR C). Factors used for calculating the pretest probability include elevated heart rate without hemoptysis, a diagnosis of cancer, recent surgery/immobilization, previous thromboembolism, and signs and symptoms of DVT. Based on these rules the patient described in the scenario has a low score and therefore a low probability of pulmonary embolism.A D-dimer level is the next most appropriate test for this low-probability scenario.

Which one of the following is diagnostic for type 2 diabetes mellitus? (check one) A. A fasting plasma glucose level ≥126 mg/dL on two separate occasions B. An oral glucose tolerance test (75-g load) with a 2-hour glucose level ≥160 mg/dL C. A random blood glucose level ≥200 mg/dL in an asymptomatic person D. A hemoglobin A1c ≥6.0% on two separate occasions

A. A fasting plasma glucose level ≥126 mg/dL on two separate occasions Current criteria for the diagnosis of diabetes mellitus include a hemoglobin A1c ≥6.5%, a fasting plasma glucose level ≥126 mg/dL, a 2-hour plasma glucose level ≥200 mg/dL, or, in a symptomatic patient, a random blood glucose level ≥200 mg/dL. In the absence of unequivocal hyperglycemia, results require confirmation by repeat testing.

An otherwise healthy 32-year-old female sees you for a routine health maintenance examination after having required laboratory screening at work. She has no significant past medical history and the physical examination is normal. Laboratory results are unremarkable except for an elevated alkaline phosphatase level.Which one of the following would be most appropriate at this point? (check one) A. A gamma-glutamyl transferase level B. A parathyroid hormone level C. Hepatitis C antibody testing D. Ultrasonography of the liver E. Bone scintigraphy

A. A gamma-glutamyl transferase level When an otherwise healthy patient's alkaline phosphatase level is elevated, it is helpful to determine whether the source of elevated isoenzymes is the liver or bone. One method is fractionation of the alkaline phosphatase by electrophoresis. A gamma-glutamyl transferase or 5 -nucleotidase level can also indicate whether the elevation of the alkaline phosphatase is from the liver. These enzymes are rarely elevated in conditions other than liver disease. Severe vitamin D deficiency can lead to osteomalacia with an elevated alkaline phosphatase.

A 41-year-old healthy female presents for a health maintenance examination. Her last preventive care visit was 3 years ago, when she had negative cervical cytology and HPV co-testing and a normal lipid panel. She does not use tobacco, alcohol, or illicit drugs. She has not been sexually active since her last visit, and her menstrual periods are regular. A complete physical examination is normal, including blood pressure.Which one of the following is recommended by the U.S. Preventive Services Task Force for this patient at this time? (check one) A. Daily folic acid supplementation B. A manual breast examination C. Chlamydia screening D. A fasting lipid profile E. A Papanicolaou test

A. Daily folic acid supplementation The U.S. Preventive Services Task Force (USPSTF) recommends that all women planning or capable of pregnancy take a daily folic acid supplement containing 0.4-0.8 mg (400-800 μg) (A recommendation). A manual breast examination by the clinician or patient is not recommended by the USPSTF. Cervical cytology (a Papanicolaou test) is recommended every 5 years if results are normal and it is combined with negative high-risk HPV testing (A recommendation). Chlamydia screening is recommended yearly for all sexually active females under age 25 and older individuals at higher risk. For women who are over 25 and not at increased risk the USPSTF makes no recommendation for or against screening (C recommendation). This patient was screened for lipid disorders 3 years ago.

A home health nurse calls you about a 62-year-old male who is recovering at home several days after spinal surgery. His recovery was going well until he became unable to urinate despite the sensation of needing to do so. His last normal void was about 12 hours ago and felt incomplete. Catheterization produced 900 mL of clear-appearing urine that she will send for urinalysis. His bowel movements have been normal and his need for pain medications has been decreasing.In addition to stopping medications that may be promoting his urinary retention, which one of the following management strategies would be most appropriate? (check one) A. Leaving the indwelling catheter in place for 48-72 hours B. Starting finasteride (Proscar), 5 mg daily C. Starting oxybutynin, 10 mg daily D. Sending the patient to the emergency department

A. Leaving the indwelling catheter in place for 48-72 hours This patient is suffering from acute urinary retention, likely due to mild benign prostatic hyperplasia exacerbated by pain medication and a lack of activity. Acute urinary retention could also possibly be due to irritation of sympathetic and/or parasympathetic nerves near the spine. Placing an indwelling bladder catheter is appropriate. It would also be reasonable, although impractical in the short term, to teach the patient or his caretakers to intermittently catheterize him.The likelihood of a successful return to voiding spontaneously will increase over time. However, the risk of catheter-associated urinary tract infection is estimated to be about 5% per day. Therefore, catheter removal and a trial of spontaneous voiding should be attempted after 48-72 hours. There is good evidence that starting an α-blocking medication such as tamsulosin during the time the catheter is in place will nearly double the success of the trial of spontaneous voiding. Finasteride in isolation is not recommended and oxybutynin would be contraindicated.

A 30-year-old female presents to your office for evaluation of a 5-mm pulmonary nodule noted on CT of the chest performed after a motor vehicle accident 2 weeks ago. She has had no symptoms and she is not a smoker. A physical examination is unremarkable.Which one of the following would be most appropriate at this point? (check one) A. Noncontrast chest CT in 1 year B. A PET scan C. Referral for a needle biopsy D. Referral for bronchoscopy E. Referral for wedge resection

A. Noncontrast chest CT in 1 year By definition, a pulmonary nodule is a circumscribed, round lesion that may measure up to 3 cm in size and is surrounded by aerated lung. Management is based on the size of the nodule and the probability of malignancy. Risk factors for lung cancer include a previous malignancy, a positive smoking history, and age ≥65. Only 1% of nodules between 2 mm and 5 mm in size are malignant.Nodules <8 mm are difficult to biopsy, and a PET scan is not reliable. The risk of surgery outweighs the benefits in nodules of this size. For a low-risk patient with a nodule 4 mm to <6 mm in size, a repeat noncontrast CT at 12 months is recommended. If it is unchanged, no further follow-up is needed.

A 45-year-old white female presents to your office after recent routine laboratory tests revealed a platelet count of 100,000/mm3 (N 150,000-350,000). Six months ago the patient's platelet count was 283,000/mm3. All other indices are within normal limits. The history is negative for easy bruising, bleeding, fever, rash, or arthralgias. She has no family history of blood disorders or recent illness. Her medications include ranitidine (Zantac), fluticasone (Flonase), and inhaled albuterol (Proventil, Ventolin). She is otherwise healthy. A peripheral smear confirms thrombocytopenia.Which one of the following would be most appropriate at this point? (check one) A. Stopping ranitidine and repeating the CBC in 2 weeks B. Oral prednisone, 50 mg daily for 7 days, and a repeat CBC in 1 week C. Outpatient transfusion of 1 unit of platelets D. Referral to a hematologist for further evaluation E. Referral for a bone marrow biopsy

A. Stopping ranitidine and repeating the CBC in 2 weeks Thrombocytopenia is a relatively common dyscrasia often discovered through routine laboratory studies. There are many causes of thrombocytopenia but medication-induced thrombocytopenia should always be considered. In this case the patient is taking an H2-blocker that may cause blood dyscrasias. The offending agent should be stopped and a repeat level should be obtained in 2-4 weeks for patients with mild asymptomatic thrombocytopenia (platelet count 100,000-150,000/mm3) and in 1-2 weeks for moderate thrombocytopenia (platelet count 50,000-100,000/mm3) (SOR C). Prednisone is thefirst-line treatment for immune thrombocytopenic purpura (SOR C); however in this situation, it is reasonable to look for other causes first.

This patient has type 2 diabetes mellitus and presents with new-onset edema in her lowerextremities. You are concerned that her symptoms and examination findings maybe related to an underlying renal pathology.To confirm your suspicion, the most appropriate diagnostic test at this time would be (check one) A. a spot urine protein to creatinine ratio B. a 24-hour urine creatinine determination C. renal ultrasonography D. renal enhanced MRI E. renal biopsy

A. a spot urine protein to creatinine ratio This patient has type 2 diabetes mellitus and presents with new-onset edema in her lowerextremities, the most common presenting symptom of nephrotic syndrome (NS). Patientswith NS may also report foamy urine, exertional dyspnea or fatigue, and significantfluid-associated weight gain. A 24-hour urine collection for protein (not creatinine) can beused to diagnose proteinuria, but the collection process is cumbersome and the specimenis often collected incorrectly. The protein-to-creatinine ratio from a single urine sample is commonly used to diagnose nephrotic-range proteinuria.

Actinic keratoses of the skin may progress to: (check one) A. nodular basal cell cancer B. pigmented basal cell cancer C. squamous cell cancer D. Merkel cell cancer E. malignant melanoma

Actinic keratoses are scaly lesions that develop on sun-exposed skin, and are believed to be carcinoma in situ. While most actinic keratoses spontaneously regress, others progress to squamous cell cancers.

During a routine health maintenance visit a 38-year-old female expresses concern about her risk of breast cancer because her mother and another relative have had breast cancer. She is asymptomatic and your clinical breast examination reveals no masses.For this patient, the U.S. Preventive Services Task Force recommends which one of the following? (check one) A. Administering a familial risk stratification tool B. BRCA mutation testing C. Bilateral screening mammography D. MRI of the breasts E. Referral for genetic counseling

Administering a familial risk stratification tool Mammographic screening is not recommended at the age of 38. The U.S. Preventive Services Task Force (USPSTF) recommends against routine mammographic screening for breast cancer between the ages of 40 and 49, but promotes a policy of individualized shared decision making. Mammography every 2 years is recommended for women between the ages of 50 and 74 (B recommendation). The USPSTF recommends any of several familial risk stratification tools for use in women who have a family member with breast, ovarian, tubal, or peritoneal cancer, to assess for an increased risk of a BRCA1 or BRCA2 mutation. If the screen is positive, a referral for genetic counseling is recommended to determine if BRCA testing is indicated (B recommendation).

A 22-year-old female who was diagnosed with bronchitis at an urgent care clinic 3 days ago sees you because her cough is still present. She is very annoyed by the cough and is concerned because she read online that she could have pneumonia. She asks if she should have a chest radiograph.Which one of the following would be an indication for a chest radiograph in this patient? (check one) A. A cough lasting more than 14 days B. A respiratory rate >24/min C. A temperature >37.5°C (99.5°F) D. Wheezing on the lung examination E. Cigarette smoking

Adult patients with acute bronchitis rarely require a chest radiograph to rule out pneumonia. Indications for a chest radiograph include dyspnea, tachypnea, tachycardia, temperature >100.0°F, bloody sputum, or signs of focal consolidation on lung auscultation. In patients with bronchitis the cough lasts an average of 18 days, so a chest radiograph would not be indicated after only 14 days. Smoking does not influence the need for a chest radiograph, and wheezing is common in uncomplicated acute bronchitis.

A 21-year-old male presents to an acute care center with pain in his left shoulder after a bicycle accident. His left arm is externally rotated and slightly abducted. A neurovascular examination is normal. Plain radiographs show an anterior shoulder dislocation. Reduction is successful, which is confirmed by a plain radiograph.Which one of the following complications would be reduced by gentle range-of-motion exercises during immobilization? (check one) A. Acromioclavicular joint injury B. Adhesive capsulitis (frozen shoulder) C. Recurrent dislocation D. Rotator cuff injury E. Shoulder impingement syndrome

After a shoulder dislocation, normal activity can resume when motion and strength in both arms is equal. Immobilization of the shoulder after a dislocation is recommended for at least 1 week. Recurrent shoulder dislocations are more common in younger patients and should be immobilized for 3 weeks in patients under 30 years of age. In patients over 30 years of age, 1 week of immobilization will limit the amount of joint stiffness. Prolonged immobilization is a risk factor for developing adhesive capsulitis (frozen shoulder). Gentle range-of-motion exercises should be performed during the immobilization period to limit the risk of adhesive capsulitis. Recurrent dislocations, rotator cuff injuries, shoulder impingement syndrome, and acromioclavicular joint injuries are not reduced by gentle range-of-motion exercises.

On two subsequent visits his blood pressure is 168/92 mm Hg and 171/91 mm Hg and you recommend treatment. The patient states that he will not take any prescription medications, as he believes they are harmful.Which one of the following nonpharmacologic measures would be appropriate to recommend for this patient? (check one) A. Limiting sodium intake to 4 g per day B. Limiting alcohol to no more than 3 drinks per day C. Daily coenzyme Q10 D. Daily magnesium supplements E. Moderate physical activity for 150 minutes or more per week

Although pharmacologic therapy is the mainstay of treatment for hypertension in adults, there are several nonpharmacologic options that have been shown to lower blood pressure. Moderate exercise 3-4 times per week for 40 minutes or more has been shown to lower high blood pressure, with the greatest effect seen when patients exercise 150 minutes or more per week. Limiting sodium intake to 2400 mg/day decreases blood pressure, and further effects are seen when it is limited to 1500 mg/day. Alcohol should be limited to no more than two drinks per day in men, and one drink per day in women. Magnesium and coenzyme Q10 do not lower blood pressure.

Which beta blockers are shown to improve mortality in patients with heart failure with a reduced ejection fraction?

Among the β-blockers, carvedilol, bisoprolol, and metoprolol succinate have this indication.

You see a 37-year-old asymptomatic male for the first time for a health maintenance visit.He underwent a splenectomy 6 years ago following a motor vehicle accident.Which one of the following would be appropriate in the care of this patient? (check one) A. Routine health care only, given that he became asplenic as an adult B. Instructing him to report any fever >101.5°F that lasts more than 72 hours C. Providing oral amoxicillin or levofloxacin (Levaquin) for initial empiric treatment of a fever if he is unable to obtain medical care within a few hours D. Forgoing influenza vaccine if inactivated vaccine is not available E. Administering 23-valent pneumococcal polysaccharide vaccine (PPSV23, Pneumovax 23) every 5 years indefinitely after the initial vaccination

Amoxicillin, levofloxacin, and moxifloxacin should be taken by asplenic patients with a new onset of fever if they cannot get to a medical facility within 2 hours for evaluation. Fever should be reported immediately due to the lifelong significant risk of sepsis. Unless otherwise contraindicated, asplenic patients should receive annual influenza immunization. Pneumococcal polysaccharide vaccine (PPSV23) should be given twice, with the second dose given 5 years after the first.

An 82-year-old white male consults you following several syncopal episodes that are clearly orthostatic in nature. During the course of your evaluation you find that he has a large tongue, mild cardiomegaly, and findings that suggest bilateral carpal tunnel syndrome.The most likely diagnosis at this time is (check one) A. pernicious anemia B. cervical spondylosis C. amyloidosis D. polymyalgia rheumatica

Amyloidosis is defined as the extracellular deposition of the fibrous protein amyloid at one or more sites. It may remain undiagnosed for years. Features that should alert the clinician to the diagnosis of primary amyloidosis include unexplained proteinuria, peripheral neuropathy, enlargement of the tongue, cardiomegaly, intestinal malabsorption, bilateral carpal tunnel syndrome, and orthostatic hypotension. Amyloidosis occurs both as a primary idiopathic disorder and in association with other diseases such as multiple myeloma.

A 25-year-old white male who has a poorly controlled major seizure disorder and a 6-week history of recurrent fever, anorexia, and persistent, productive coughing visits your office. On physical examination he is noted to have a temperature of 38.3°C (101.0°F), a respiratory rate of 16/min, gingival hyperplasia, and a fetid odor to his breath. Auscultation of the lungs reveals rales in the mid-portion of the right lung posteriorly.Which one of the following is most likely to be found on a chest radiograph? (check one) A. Sarcoidosis B. Miliary calcifications C. A lung abscess D. A right hilar mass E. A right pleural effusion

Anaerobic lung abscesses are most often found in a person predisposed to aspiration who complains of a productive cough associated with fever, anorexia, and weakness. Physical examination usually reveals poor dental hygiene, a fetid odor to the breath and sputum, rales, and pulmonary findings consistent with consolidation.

A 67-year-old female has started receiving home hospice care. Her attending physician can bill through which one of the following? (check one) A. Medicare Part A B. Medicare Part B C. Medicare Part C D. Medicare Part D E. The attending physician cannot bill Medicare

As long as the attending physician is not employed by hospice, Medicare Part B can be billed. -Medicare Part A (hospital insurance) covers inpatient care in hospitals and skilled nursing facilities, hospice, and home health services, but not custodial or long-term care. -Medicare Part B (medical insurance) covers outpatient physician services, including office visits and home health services. -Medicare Part C (Medicare Advantage Plans) is offered by private companies, and combines Part A and Part B coverage. These plans always cover emergency and urgent care, and may offer extra coverage such as vision, hearing, dental, and/or health and wellness programs. -Most plans also include Medicare Part D, which provides prescription drug coverage. Medicare Part D plans vary with regard to cost and drugs covered.

A 26-year-old female presents with a 3-month history of abdominal pain, lightheadedness with standing, and some hyperpigmentation. Her CBC is normal, but a basic metabolic panel reveals a slightly low sodium level and a slightly high potassium level.Which one of the following would be the most appropriate next step in your evaluation of this patient? (check one) A. A serum aldosterone level B. A corticotropin (ACTH) stimulation test C. Paired morning cortisol and melanocyte stimulating hormone levels D. 21-hydroxylase antibodies and 17-hydroxyprogesterone levels

B. A corticotropin (ACTH) stimulation test This patient has signs and symptoms consistent with primary adrenal insufficiency (PAI). In Western countries autoimmunity is responsible for 90% of these cases. Because the corticotropin (ACTH) stimulation test has a higher degree of sensitivity and specificity than morning cortisol and ACTH concentrations, it is the preferred test in all patients with possible primary adrenal insufficiency.

A 42-year-old female is found to have a thyroid nodule during her annual physical examination. Her TSH level is normal. Ultrasonography of her thyroid gland shows a solitary nodule measuring 1.2 cm.Which one of the following would be most appropriate at this point? (check one) A. A radionuclide thyroid scan B. A fine-needle aspiration biopsy of the nodule C. Partial thyroidectomy D. Total thyroidectomy E. Reassurance

B. A fine-needle aspiration biopsy of the nodule All patients who are found to have a thyroid nodule on a physical examination should have their TSH measured. Patients with a suppressed TSH should be evaluated with a radionuclide thyroid scan; nodules that are "hot" (show increased isotope uptake) are almost never malignant and fine-needle aspiration biopsy is not needed. For all other nodules, the next step in the workup is a fine-needle aspiration biopsy to determine whether the lesion is malignant (SOR B).

This patient has classic signs and symptoms of viral bronchiolitis, likely due to respiratory syncytial virus (RSV). A chest radiograph is not indicated in a patient with a classic presentation and no focal findings on examination. Most concerning is his history of low urine output, suggesting inadequate oral intake. This is often related to a high respiratory rate and copious nasal secretions In addition to oral or intravenous rehydration, which one of the following treatment plans is most appropriate for this patient at this time? (check one) A. Send the child home and follow up tomorrow B. Admit to the hospital for supportive care only C. Admit to the hospital for inhaled bronchodilators D. Admit to the hospital for inhaled bronchodilators and oral dexamethasone E. Admit to the hospital for inhaled bronchodilators, oral dexamethasone, and intravenous antibiotics

B. Admit to the hospital for supportive care only The patient requires hospitalization for monitoring of his respiratory status and supportive care, including intravenous or nasogastric rehydration. At this time the infant does not require supplemental oxygen, as his oxygen saturation is above 90%. Many medications have been studied for the treatment of bronchiolitis in children and most have been found to not provide benefit with regard to the need for hospitalization, length of hospitalization, or disease resolution. Medications that are NOT recommended include inhaled bronchodilators, inhaled epinephrine, inhaled or systemic corticosteroids, and antibiotics.

A 14-year-old female with mild depression sees you for follow-up. After her last visit she began a trial of psychotherapy but her mother asks about additional treatment options. The daughter has no other significant past medical history.Which one of the following is recommended as first-line pharmacotherapy for this patient? (check one) A. Citalopram (Celexa) B. Fluoxetine (Prozac) C. Mirtazapine (Remeron) D. Paroxetine (Paxil) E. Venlafaxine (Effexor XR)

B. Fluoxetine (Prozac) Expert panel guideline recommendations suggest fluoxetine should be the first-line pharmacotherapy option for adolescents after a trial of psychotherapy. The patient should be monitored weekly for side effects for a month after starting fluoxetine. If fluoxetine is ineffective, sertraline and citalopram are recommended as alternatives. Venlafaxine should be avoided in adolescents because it is associated with a statistically increased risk of suicidal behavior or ideation.

A 43-year-old female presents with marked proximal muscle weakness, dysphagia, and pain in the shoulders and hips, all beginning within the past 5 weeks. She reports difficulty getting out of a chair. On examination she has a violaceous rash involving the periorbital skin, and macular erythematous lesions over the anterior chest and upper lateral thighs.Which one of the following additional findings would you expect? (check one) A. Hyperkeratotic plaques in intertriginous areas B. Macules over the extensor surfaces of her joints C. Polygonal papules on the flexor surface of her wrists D. Distal onycholysis E. Atrophic cuticles with contracted nail-fold capillaries on dermoscopy

B. Macules over the extensor surfaces of her joints This patient's symptoms and findings suggest dermatomyositis. This disease is distinguished from autoimmune myopathies and polymyositis by distinct dermatologic findings, including Gottron's sign (nonpalpable macules over the extensor surface of joints). Patients may also have dilated nail-fold capillaries and ragged, thickened cuticles. Distal onycholysis is most commonly associated with onychomycosis, while hyperkeratotic plaques are not a feature of dermatomyositis. Polygonal papules on the wrist flexor surfaces are seen in lichen planus.

A 16-year-old female who plays competitive soccer develops anterior knee pain that is worse with downhill running and after prolonged sitting. An examination shows no effusion or instability, no joint line tenderness, an increased Q-angle, and a negative McMurray's test. A knee radiograph is negative.Which one of the following is the most likely diagnosis? (check one) A. Osgood-Schlatter syndrome B. Patellofemoral pain syndrome C. Pes anserine bursitis D. Prepatellar bursitis E. A torn medial meniscus

B. Patellofemoral pain syndrome Patellofemoral pain syndrome is a common cause of anterior knee pain, especially in women. It is worse with running downhill or going down stairs. It is not associated with a knee effusion. The examination is often positive for an apprehension test over the patella.

A 45-year-old female. As you perform a pelvic examination you notice a fullness in the left adnexal region that is mildly tender to palpation. The remainder of the examination is normal. You order pelvic ultrasonography, which demonstrates a 4×5-cm cystic structure on the left ovary. There are no internal septations or echogenic internal structures. A pregnancy test is negative.Which one of the following would be most appropriate at this point? (check one) A. Reassurance only B. Repeat pelvic ultrasonography in 8-10 weeks C. A serum CA-125 level D. An oral contraceptive and repeat pelvic ultrasonography in 6-8 months E. Referral to a gynecologist for diagnostic cystectomy

B. Repeat pelvic ultrasonography in 8-10 weeks Functional ovarian cysts are estimated to be present in approximately 15% of menstruating women and are often found incidentally on a pelvic examination or on diagnostic imaging performed for other reasons. The majority of these lesions resolve spontaneously within two or three menstrual cycles and management should consist of follow-up ultrasonography 8-12 weeks after the cyst is identified. Cysts that are particularly symptomatic, have less than simple features, are >10 cm in size, or that persist longer than 12 weeks should be referred for consideration of diagnostic removal (cystectomy or oophorectomy depending on patient characteristics)

You see a 12-year-old female for a well child check. She is healthy without any medical problems and neither she nor her father have any concerns today. She is up to date on her immunizations except for her third dose of HPV vaccine. She received the first dose of the vaccine at her 11-year-old well child check and her second dose 1 month later. However, she was told by a few friends that they only needed to get two doses of HPV vaccine, so she is very excited that she does not need any shots today.According to the CDC, which one of the following is true regarding HPV vaccine for this patient? (check one) A. She has completed her HPV vaccine series B. She should receive a third dose of HPV vaccine today C. She needs an HPV booster at age 21 but does not need a third dose of vaccine today D. She should have HPV titers drawn today and receive a third dose of vaccine only if the titers are low

B. She should receive a third dose of HPV vaccine today In 2016 the CDC changed the recommendation for the number of HPV vaccine doses for children ages 11-14. Children in this age group need only two doses of HPV vaccine 6-12 months apart. However, if they received two doses of HPV vaccine less than 5 months apart, they still need to have the third dose. Children and young adults over the age of 14 and those with certain immunocompromising conditions still require three doses of HPV vaccine. There is no indication for a booster dose at a later date, nor is there clinical data to support using titers to gauge immunogenicity to HPV.

A 67-year-old male is admitted to the intensive-care unit with complications of sarcoma of the leg, including multiple pulmonary emboli. He most likely will require an urgent above-the-knee amputation.A spiritual assessment in this patient (check one) A. should be delegated to the hospital chaplain B. can be performed by the physician C. is not appropriate if the patient has not listed a religion in his demographics D. should be postponed until the patient is in less critical condition

B. can be performed by the physician The Joint Commission for Hospital Accreditation now requires a patient spiritual assessment upon hospital admission. Using the FICA Spiritual History Tool or HOPE questions for making the spiritual assessment is appropriate for the physician. HOPE is a mnemonic for sources of Hope, Organized religion, Personal spirituality and practices, and Effects on medical care and end-of-life issues. The FICA tool includes questions in the categories of Faith and beliefs, Importance, Community, and how to Address these issues when providing care. It is very appropriate for a physician to conduct a spiritual assessment in older, hospitalized patients with critical or terminal illnesses. Some patients may consider themselves spiritual but not necessarily religious.

A 13-year-old male is brought to your office because of pain in his foot. Two days ago he stepped on a nail that went through his sneaker and caused a puncture wound to the base of his foot. On examination today he has tenderness and erythema surrounding the wound, and you can express pus from the wound. He is afebrile.Which one of the following would be best to treat this patient's cellulitis? (check one) A. Amoxicillin/clavulanate (Augmentin) B. Cephalexin (Keflex) C. Ciprofloxacin (Cipro) D. Doxycycline E. Trimethoprim/sulfamethoxazole (Bactrim)

C. Ciprofloxacin (Cipro) Puncture wounds to the foot commonly get infected. Most soft-tissue infections from puncture wounds are caused by gram-positive organisms. Staphylococcus aureus is the most common, followed by other staphylococcal and streptococcal species. When the puncture wound is through the rubber sole of an athletic shoe, Pseudomonas is the most frequent pathogen. Ciprofloxacin is the only oral antibiotic that has antipseudomonal activity, and would be the most appropriate choice.

Which one of the following should be given intravenously in the initial treatment of status epilepticus? (check one) A. Propofol (Diprivan) B. Phenobarbital C. Lorazepam (Ativan) D. Midazolam (Versed)

C. Lorazepam (Ativan) Status epilepticus refers to continuous seizures or repetitive, discrete seizures with impaired consciousness in the interictal period. It is an emergency and must be treated immediately, since cardiopulmonary dysfunction, hyperthermia, and metabolic derangement can develop, leading to irreversible neuronal damage. Lorazepam, 0.1-0.15 mg/kg intravenously, should be given as anticonvulsant therapy after cardiopulmonary resuscitation. This is followed by phenytoin, given via a dedicated peripheral intravenous line. Fosphenytoin, midazolam, or phenobarbital can be used if there is no response to lorazepam.Propofol has been used for refractory status epilepticus to induce general anesthesia when the initial drugs have failed, but reports of fatal propofol infusion syndrome have led to a decline in its use.

A 42-year-old male comes to your office with acute right flank pain that awakened him from sleep. The pain is colicky and he says it is the most intense pain that he has ever felt. The findings on a physical examination, in addition to blood on his urinalysis, make you suspect a urinary tract stone.Which one of the following imaging modalities would be most appropriate for confirming your suspicion? (check one) A. Abdominal radiography (KUB) B. Standard CT of the abdomen and pelvis with intravenous contrast C. Low-dose helical (spiral) noncontrast CT of the abdomen and pelvis D. MRI of the abdomen and pelvis without contrast E. Ultrasonography of the kidneys and bladder

C. Low-dose helical (spiral) noncontrast CT of the abdomen and pelvis

A 45-year-old male is seen in the emergency department with a 2-hour history of substernal chest pain. An EKG shows an ST-segment elevation of 0.3 mV in leads V4-V6. In addition to evaluation for reperfusion therapy, which one of the following would be appropriate? (check one) A. Enteric aspirin, 81 mg B. Intravenous metoprolol (Lopressor) C. Oral clopidogrel (Plavix) D. Warfarin (Coumadin), after blood is drawn to establish his baseline INR E. Delaying treatment pending results of two sets of cardiac enzyme measurements

C. Oral clopidogrel (Plavix) This patient has an ST-segment elevation myocardial infarction (STEMI). STEMI is defined as an ST-segment elevation of greater than 0.1 mV in at least two contiguous precordial or adjacent limb leads. The most important goal is to begin fibrinolysis less than 30 minutes after the first contact with the health system. The patient should be given oral clopidogrel, and should also chew 162-325 mg of aspirin.Enteric aspirin has a delayed effect. Intravenous β-blockers such as metoprolol should not be routinely given, and warfarin is not indicated. Delaying treatment until cardiac enzyme results are available in a patient with a definite myocardial infarction is not appropriate.

A 67-year-old male is admitted to your inpatient service with a week-long acute exacerbation ofCOPD. He also has hypertension and type 2 diabetes mellitus. After 24 hours of intravenous fluids and intravenous methylprednisolone, he is now tolerating oral intake.Which one of the following corticosteroid regimens is best for this patient at this time? (check one) A. Continue intravenous methylprednisolone until his COPD is back to baseline, then switch to oral methylprednisolone for a 14-day total course of treatment B. Switch to oral prednisone for a 14-day total course of treatment, including the initial 24-hour intravenous treatment C. Switch to oral prednisone for 4 more days of treatment D. Use only inhaled corticosteroids by nebulizer E. Discontinue corticosteroid treatment altogether after 24 hours

C. Switch to oral prednisone for 4 more days of treatment Systemic corticosteroid therapy reduces the hospital length of stay in patients with acute COPD exacerbations (SOR A). Oral therapy has been shown to be as effective as the intravenous route in patients who can tolerate oral intake (SOR B). A randomized, controlled trial has demonstrated that 5-day coursesof systemic corticosteroid therapy are at least as effective as 14-day courses (SOR A). Inhaled corticosteroids are beneficial in some COPD patients but nebulizers generally do not offer significant advantages over metered-dose inhalers in most patients.

A 47-year-old female is concerned about a change in her menstrual pattern. Her monthly periods continue, but for the past several months they have been heavier than usual and have been lasting a few days longer. Last month she also noted some spotting for several days prior to the onset of her menses. Her pelvic examination is normal.Which one of the following would be most appropriate at this time? (check one) A. Observation only, and reexamination in 3 months B. A serum FSH level C. Transvaginal ultrasonography D. Progestin-only therapy to normalize bleeding E. Cyclic estrogen-progestin therapy to normalize bleedin

C. Transvaginal ultrasonography Abnormal uterine bleeding can be a sign of endometrial cancer in premenopausal women, who account for 20% of cases of endometrial cancer. The American College of Obstetricians and Gynecologists recommends that women with abnormal uterine bleeding should be evaluated for endometrial cancer if they are older than 45 years or if they have a history of unopposed estrogen exposure (SOR C). Most guidelines recommend either transvaginal ultrasonography or endometrial biopsy as the initial study in the evaluation of endometrial cancer. Transvaginal ultrasonography is often preferred as the initial study because of its availability, cost-effectiveness, and high sensitivity. If bleeding persists despite normal transvaginal ultrasonography a tissue biopsy should be performed.

An 82-year-old male presents to the emergency department with severe generalized abdominal pain. He has a history of paroxysmal atrial fibrillation and stopped taking rivaroxaban (Xarelto) 2 months ago because of the cost. The physical examination reveals generalized abdominal tenderness and an epigastric bruit. You suspect mesenteric ischemia.Which one of the following is the recommended imaging study? (check one) A. Ultrasonography B. Endoscopy C. Catheter angiography D. CT angiography E. MR angiography

CT angiography (CTA) is the recommended imaging modality for the diagnosis of visceral ischemic syndromes because of its 95%-100% accuracy. Images of the origins and length of the vessels can be obtained rapidly, characterize the extent of stenosis or occlusion and the relationship to branch vessels, and aid in the assessment of options for revascularization. Endoscopy is most useful for diagnosing conditions other than mesenteric ischemia.

His medications include metformin (Glucophage), glipizide (Glucotrol XL), lisinopril (Prinivil, Zestril), aspirin, simvastatin (Zocor), and fluoxetine (Prozac). His BMI is 52.4 kg/m2 and he is struggling to lose weight.Which one of the following medication replacements could help promote weight loss? (check one) A. Atorvastatin (Lipitor) instead of simvastatin B. Canagliflozin (Invokana) instead of glipizide C. Carvedilol (Coreg) instead of lisinopril D. Paroxetine (Paxil) instead of fluoxetine E. Pioglitazone (Actos) instead of glipizide

Canagliflozin (Invokana) instead of glipizide This patient is taking medications that help with weight loss (metformin) and medications that are weight neutral (lisinopril, simvastatin, and fluoxetine). Glipizide, however, causes weight gain, and switching to an SLGT2 inhibitor such as canagliflozin can help promote weight loss. Likewise, the patient could use a GLP-1 receptor agonist such as exenatide or an amylin mimetic (pramlintide) for weight loss benefits. Sulfonylureas, thiazolidinediones, and insulins all promote weight gain (SOR A). Fluoxetine and sertraline are weight neutral, whereas paroxetine can cause weight gain (SOR B).

Which histologic type of colonic polyps >1.0 cm in size has the highest likelihood of becoming malignant? (check one) A. Hamartomatous polyps B. Hyperplastic polyps C. Inflammatory polyps D. Tubular adenomas E. Villous adenomas

Colon cancer arises from adenomatous polyps, and generally requires at least 5 years of growth before malignant transformation. Villous adenomas carry a threefold increased risk for becoming malignant compared with other adenomatous types such as tubular or tubulovillous adenomas. The larger the polyp the greater the chance of malignancy, although malignant polyps <1.5 cm are rare. Hamartomas (juvenile polyps) and inflammatory polyps (often associated with inflammatory bowel disease) are benign. Hyperplastic polyps are the most common histologic type by far, but only rarely become cancerous.

Opioid therapies provide the greatest analgesic relief for most patients with a terminal illness. However, concerns about which one of the following can inappropriately limit the use of opioids in these patients? (check one) A. Angina B. Dementia C. Gastritis D. Renal failure E. Respiratory depression

Concerns about addiction and respiratory depression often limit the use of opioids or lead to inadequate dosages in patients with a terminal illness who are experiencing pain at the end of life (SOR C)

A 22-year-old white female comes to your office for a routine health maintenance examination. She has a BMI of 27.0 kg/m2 and a blood pressure of 113/78 mm Hg. She has no significant past medical history or family history and the examination is normal.According to U.S. Preventive Services Task Force guidelines, this patient should be screened for diabetes mellitus beginning at age (check one) A. 25 B. 30 C. 35 D. 40 E. 45

D. 40 According to U.S. Preventive Services Task Force guidelines, adults age 40-70 should be screened for diabetes mellitus if they are overweight (BMI >25.0 kg/m2) or obese (BMI ≥30.0 kg/m2), so this patient should not be screened until age 40. Screening for diabetes mellitus should be considered for any adult who has a risk factor such as a family history of diabetes, a personal history of gestational diabetes, polycystic ovary syndrome, or being a member of a high-risk ethnic group (African-American, Hispanic, American Indian, Alaskan Native, or Native Hawaiian).

A 38-year-old female with diabetes mellitus controlled by diet has a sodium level of 130 mEq/L (N 136-145) on a routine basic metabolic panel. She does not use any dietary supplements and is not taking a diuretic. You consider a diagnosis of syndrome of inappropriate secretion of antidiuretic hormone (SIADH) and take additional history and order additional laboratory studies.Which one of the following would be most consistent with SIADH? (check one) A. A history of excessive beer drinking B. A history of polydipsia C. A serum glucose level of 350 mg/dL (N 70-100) D. A urine sodium level of 40 mEq/L (N 20) E. A urine osmolality of 90 mOsm/kg (N 300-900)

D. A urine sodium level of 40 mEq/L (N 20) A diagnosis of the syndrome of inappropriate secretion of antidiuretic hormone (SIADH) generally starts with the discovery of hyponatremia and is confirmed after all other possible causes are excluded. Inappropriate release of ADH increases free water reabsorption, which increases circulating blood volume, dilutes sodium, and lowers hematocrit and hemoglobin. Urine output is often lowered because of this reabsorption, and the urine is more concentrated (urine osmolality > plasma osmolality) with sodium levels >20 mEq/L. Modest weight gain may be noted as a result of the increased blood volume.

A 60-year-old male presents for evaluation of mild pitting edema of both lower extremities for several months. He has daytime fatigue and drowsiness but no orthopnea or paroxysmal nocturnal dyspnea. His only medications are hydrochlorothiazide and lisinopril (Prinivil, Zestril). In addition to a sleep study, which one of the following would be the most appropriate next step in the evaluation of this patient? (check one) A. An ankle-brachial index B. A D-dimer assay C. Lymphoscintigraphy D. Echocardiography E. Magnetic resonance venography

D. Echocardiography When evaluating bilateral lower extremity edema, one should first look for systemic etiologies that would result in edema, such as hepatic, renal, or cardiac failure. In patients with obesity or a history of loud snoring, daytime drowsiness, or unrestful sleep, obstructive sleep apnea is likely. These patients can be diagnosed through polysomnography. Echocardiography is also recommended to detect pulmonary hypertension.

At a routine visit, a 40-year-old female asks about beginning an exercise regimen. She has a family history of heart disease and hypertension. She currently has no medical problems, but she is sedentary.Which one of the following would be the most appropriate recommendation for this patient? (check one) A. A baseline EKG and rhythm strip B. An exercise stress test prior to beginning exercise C. Jogging for 30 minutes twice a week D. Fast walking for 30 minutes 5 or more days per week

D. Fast walking for 30 minutes 5 or more days per week This patient would benefit from exercise to prevent or delay the onset of heart disease and hypertension, and to manage her weight. Exercise stress testing is not specifically indicated for this patient. Current recommendations are for healthy adults to engage in 30 minutes of accumulated moderate-intensity physical activity on 5 or more days per week.

A 60-year-old male presents with a 6-week history of worsening bilateral shoulder, upper arm, and neck pain. He has morning stiffness that lasts at least an hour. The review of systems is otherwise negative. There is no localized tenderness or motor weakness on physical examination. His erythrocyte sedimentation rate is 55 mm/hr.Which one of the following is the best treatment option for this patient at this time? (check one) A. Aspirin, 1000 mg 3 times daily B. Indomethacin, 25-50 mg 3 times daily C. Methotrexate, 7.5 mg once a week D. Prednisone, 10-20 mg once daily E. Prednisone, 20 mg 3 times daily

D. Prednisone, 10-20 mg once daily The patient described has polymyalgia rheumatica (PMR). The hallmark of this condition is the rapid and often dramatic response, typically within a few days, to low-dose corticosteroids. In fact, the lack of response to low-dose prednisone in such a case should prompt the physician to consider another diagnosis.

A 23-year-old male presents with a lump in his left testicle that he found while showering last week. He has a history of orchiopexy for cryptorchidism at age 17. He is otherwise healthy. Testicular ultrasonography reveals a hypoechoic mass in his left testicle.Which one of the following would be most appropriate at this time? (check one) A. Watchful waiting B. Serum β-hCG, α-fetoprotein, and LDH levels C. CT of the abdomen and pelvis D. Referral to a urologist

D. Referral to a urologist Patients with a history of cryptorchidism are at high risk for the development of testicular cancer, especially if orchiopexy is performed after puberty. If sonography shows a hypoechoic mass, a testicular biopsy is contraindicated, since it may contaminate the scrotum or alter the lymphatic drainage. Radical inguinal orchiectomy is both diagnostic and therapeutic. Watchful waiting would not be an option in this high-risk patient. CT of the chest, abdomen, and pelvis, and measurement of the tumor markers are useful for staging and as an indication of tumor burden, but they are not diagnostic.

A 64-year-old male presents with a 2-week history of a worsening constant burning pain in his right foot. The physical examination reveals an absent dorsalis pedis pulse, and pallor develops with elevation of the foot. The resting right ankle-brachial index is 0.45 and Doppler waveform analysis indicates an isolated severe stenosis of the right posterior tibial artery.Which one of the following therapeutic interventions would be most appropriate at this point? (check one) A. A supervised walking program B. Cilostazol (Pletal) C. Warfarin (Coumadin) D. Revascularization

D. Revascularization Medication and/or a walking program have been shown to improve functional capacity in patients with symptomatic peripheral artery disease (PAD). However, this patient has critical limb ischemia and needs urgent revascularization. Endovascular therapy of isolated disease below the knee is not recommended. These patients should undergo femoral-tibial bypass. Warfarin is not recommended for the treatment of PAD.

A 40-year-old female presents with a 4-week history of a persistent sore throat despite supportive treatment for a viral upper respiratory infection provided by an urgent care facility. She reports palpitations, weight loss, frequent bowel movements, and anxiety with insomnia for the past month.On examination she has a mildly enlarged thyroid gland. Laboratory evaluation is notable for a suppressed TSH level along with elevated free T4 and total T3 levels. A radioactive iodine uptake scan shows low uptake.Which one of the following is the most likely diagnosis? (check one) A. Factitious thyrotoxicosis B. Graves disease C. Multinodular goiter D. Subacute thyroiditis E. TSH-secreting pituitary adenoma

D. Subacute thyroiditis The initial "destructive" phase of subacute thyroiditis presents with signs, symptoms, and laboratory findings of overt hyperthyroidism; however, a radioactive iodine uptake scan is negative in this phase. Graves disease and toxic multinodular goiter also present with overt hyperthyroidism, but radioactive iodine uptake is high. Factitious thyrotoxicosis is associated with low TSH and elevated or normal free T4 and total T3, but a goiter is not present. A TSH-secreting pituitary adenoma results in elevated TSH, free T4, and total T3.

You are called by the parents of a 6-year-old male because he has a 2-week history of awakening at night with severe back pain. You request an immediate evaluation in your office.A likely cause of this pain is (check one) A. rheumatoid arthritis B. lumbar sprain C. compression fracture D. discitis E. scoliosis

D. discitis Back pain that regularly occurs at night and awakens a child is usually associated with tumors or infections, such as osteomyelitis, diskitis, osteoid osteoma, osteoblastoma, and spinal cord tumors. Other possible symptoms associated with nighttime back pain include fever, malaise, and weight loss. Back pain that occurs at night is an indication for immediate medical evaluation.

Patient with cirrhosis. The physical examination is notable for mild ascites and trace lower extremity edema.Which one of the following recommendations would be most appropriate for this patient? (check one) A. Fluid restriction B. Limiting sodium intake C. A low-protein diet D. Discontinuation of metoprolol E. Discontinuation of simvastatin

Dietary counseling is key in the management of cirrhosis. Patients with ascites should be limited to 2000 mg of sodium daily (SOR A). Fluid restriction is not recommended unless the serum sodium level is <120 mEq/L, and it is extremely difficult to achieve. Patients would need to limit fluid intake to less than their urinary output, which is often reduced. Up to 60% of patients with cirrhosis suffer from malnutrition, so a high-protein diet is recommended (1.0-1.5 g/kg dry body weight). High-protein diets are tolerated well and result in improved mental status (SOR B).

Which one of the following has the best evidence supporting its use for acute low back pain without radicular symptoms? (check one) A. Acupuncture B. Bed rest C. Lumbar support D. Oral corticosteroids E. Cyclobenzaprine

E. Cyclobenzaprine NSAIDs, acetaminophen, and muscle relaxants are effective for the treatment of acute low back pain (SOR A). There is moderate-quality evidence that nonbenzodiazepine muscle relaxants are beneficial in the treatment of acute low back pain. There is also moderate-quality evidence that NSAIDs combined with nonbenzodiazepine muscle relaxants may have additive benefit for decreasing pain. Bed rest is not helpful in the treatment of acute back pain and is not recommended (SOR A).

Dude gets a splinter. After thorough wound cleansing, you inquire about his tetanus status. He is certain that he received all of his primary childhood vaccines and a "tetanus booster" at age 20, but does not know which vaccine he received.Which one of the following is the best choice for this patient regarding tetanus immunization at this time? (check one) A. TT (tetanus toxoid) B. Td (tetanus toxoid with reduced diphtheria) C. Tdap (tetanus toxoid with reduced diphtheria and acellular pertussis) D. TIG (tetanus immune globulin) E. No immunization

E. No immunization Since 2005,the recommendation for tetanus prophylaxis has included coverage not only for diphtheria (Td) but also pertussis, due to waning immunity in the general population. The current recommendation for adults who require a tetanus booster (either as a routine vaccination or as part of treatment for a wound) is to use the pertussis-containing Tdap unless it has been less than 5 years since the last booster in someone who has completed the primary vaccination series. In this scenario, no additional vaccination is needed at this time, since the patient is certain of completing the primary vaccinations and received a tetanus booster within the previous 5 years. Had the interval been longer than 5 years, then a single dose of Tdap would be appropriate unless his previous booster was Tdap. Tetanus immune globulin is recommended in addition to tetanus vaccine for wounds that are tetanus-pronedue to contamination and tissue damage in persons with an uncertain primary vaccine history. Plain tetanustoxoid (TT) is usually indicated only when the diphtheria component is contraindicated, which isuncommon.

An otherwise healthy 64-year-old male presents with mild shortness of breath and lightheadedness with exertion. He has a grade 3/6 systolic ejection murmur at the right second intercostal space radiating to the neck. A transthoracic echocardiogram and cardiac catheterization with normal coronary arteries reveals severe aortic stenosis.Which one of the following would be the most appropriate management of this patient? (check one) A. No treatment, and monitoring for increasing symptoms every 6 months B. No treatment, and monitoring with echocardiography every 6 months C. Medical management with an ACE inhibitor and continued monitoring D. Medical management with a β-blocker and continued monitoring E. Prompt aortic valve replacement

E. Prompt aortic valve replacement This patient should have his aortic valve replaced. He meets criteria for severe aortic stenosis with a transthoracic velocity ≥4.0 m/sec and an aortic valve area <1.0 cm2. Symptomatic patients with severe aortic stenosis have 2-year mortality rates of more than 50%. After valve replacement the 10-year survival rate is almost identical to that of patients without aortic stenosis. Watchful waiting with monitoring for symptoms and periodic echocardiograms is indicated for asymptomatic patients with moderate to severe aortic stenosis who have a normal ejection fraction. There is no medical treatment that delays the progression of aortic valve disease or improves survival. Measures to reduce cardiovascular risk, including treatment of hypertension, are indicated. Rate-slowing calcium channel blockers and β-blockers that depress left ventricular function should be avoided if possible. ACE inhibitors may improve symptoms in patients with aortic stenosis who are not surgical candidates.

A 37-year-old female who smokes 1 pack of cigarettes per day has just had her third child and requests contraception. She does not intend to have any more children.Which one of the following is the safest option for this patient? (check one) A. Traditional combined oral contraceptive pills B. Extended-cycle combined oral contraceptive pills C. The contraceptive patch (Ortho Evra) D. The contraceptive vaginal ring (NuvaRing) E. The etonogestrel implant (Nexplanon)

E. The etonogestrel implant (Nexplanon) Cigarette smoking, increasing age, and exogenous estrogen, particularly at the supraphysiologic doses used in contraceptives, all increase risk for vascular events such as venous thromboembolism and stroke. The use of estrogen-containing contraception in smokers ≥35 years is contraindicated because of this risk. All of the contraceptive options listed contain estrogen except for the etonogestrel implant. Other progestin-only contraceptive options that could be considered include depot medroxyprogesterone acetate, the levonorgestrel-releasing IUD, and progestin-only oral contraceptive pills.

A 38-year-old female who recently underwent a laparoscopic sleeve gastrectomy for weight loss presents to your office for a follow-up visit. She has had no complications with her recent postoperative course. Her medical history includes diabetes mellitus, hypertension, and hyperlipidemia. She has also had problems with bilateral knee pain from osteoarthritis.Patient education should include advising the patient to (check one) A. drink extra fluids with meals B. increase her intake of fibrous vegetables C. avoid pregnancy for 3 years D. take ibuprofen as needed for pain E. have a bone density test in 2 years

E. have a bone density test in 2 years After bariatric surgery the patient's postoperative medications may require adjustments and NSAIDs should be avoided. Patients should be encouraged to eat three meals and one or two snacks daily. Very dry foods, bread, and fibrous vegetables are most likely to cause problems. Fluids should be avoided during meals and for 15-30 minutes before and after meals.Those desiring pregnancy should wait 12-18 months after surgery. Recommended laboratory studies include a CBC, a metabolic profile, a folic acid level, iron studies, a parathyroid hormone level, a lipid profile, vitamin B12 levels, 24-hour urinary calcium excretion, and 25-hydroxyvitamin D levels. It is recommended that bone density measurements be done every 2 years.

A 50-year-old female comes to your office for routine health maintenance. She jogs 2 miles a day and has had left medial knee pain for the last 6 months. Radiographs reveal moderate degenerative arthritis of the knee. Her BMI is 24.1 kg/m2 and her physical examination, including an examination of the knee, is otherwise normal.Which one of the following would be most effective for this patient's arthritis? (check one) A. Weight loss B. A knee brace C. Foot orthoses D. Hyaluronic acid injection E. Exercise-based physical therapy

Exercise-based therapy is the foundation for treating knee osteoarthritis. Foot orthoses can be helpful for anterior knee pain but this patient's pain is located medially. The benefit of hyaluronic acid injections is controversial, and recommendations vary; recent systematic reviews do not support a clinically significant benefit. Weight loss is recommended for patients with a BMI >25.0 kg/m2. Wearing a knee brace has shown little or no benefit for reducing pain or improving knee function.

Atrial fibrillation is the most common cardiac arrhythmia and can be a source of morbidity and mortality. How do you confirm the diagnosis? Which one of the following would be the best choice to control this patient's heart rate? (he has COPD) (check one) A. Cardioversion B. Amiodarone (Cordarone) C. Digoxin (Lanoxin) D. Diltiazem (Cardizem) E. Propranolol

If this is suspected, a 12-lead EKG should be obtained to confirm the diagnosis. Patients need evaluation for possible cardioversion, rate versus rhythm control, and anticoagulation. The first-line agent to achieve a target heart rate of <80 beats/min would be either a β-blocker or a nondihydropyridine calcium channel blocker. This patient has significant COPD, which eliminates the use of a nonselective β-blocker such as propranolol. A nondihydropyridine calcium channel blocker such as verapamil or diltiazem would be a better choice.

When is biopsy warranted for a patient with a lymph node?

Immediate biopsy is warranted if the patient does not have inflammatory symptoms and the lymph node is >3 cm, if the node is in the supraclavicular area, or if the patient has coexistent constitutional symptoms such as night sweats or weight loss. Immediate evaluation is also indicated if the patient has risk factors for malignancy. Treatment with antibiotics is warranted in patients who have inflammatory symptoms such as pain, erythema, fever, or a recent infection.

A 65-year-old female develops gram-negative septicemia from a urinary tract infection. Despite the use of fluid resuscitation she remains hypotensive, with a mean arterial pressure of 50 mm Hg.Which one of the following would be the most appropriate treatment for this patient? (check one) A. Vasopressin B. Phenylephrine (Neo-Synephrine) C. Epinephrine D. Norepinephrine (Levophed) E. Low-dose dopamine

In a patient with sepsis, vasopressors are indicated when fluid resuscitation does not restore organ perfusion and blood pressure. Norepinephrine and dopamine are the preferred pressor agents; however, norepinephrine appears to be more effective and has a lower mortality rate. Norepinephrine is the preferred drug for shock due to sepsis. Its relative safety suggests that it be used as an initial vasopressor. It is a potent vasoconstrictor and inotropic stimulant and is useful for shock. As a first-line therapy norepinephrine is associated with fewer adverse events, including arrhythmia, compared to dopamine.

A 4-year-old female is brought in by her mother because of a 3-day history of left ear drainage and a low-grade fever. The patient has bilateral ventilation tubes that were placed 2 months ago.Which one of the following would be most effective for resolving this child's condition? (check one) A. Saline rinses of the affected ear canal B. Oral amoxicillin/clavulanate (Augmentin) C. Ciprofloxacin eardrops (Cetraxal) D. Fluocinonide eardrops

In patients who present with an ear discharge and have ventilation tubes in place, antibiotic eardrops (with or without corticosteroids) can resolve the discharge and improve the illness-related quality of life more quickly than oral antibiotics, corticosteroid eardrops, or saline rinses. The antibiotic eardrops of choice are fluoroquinolones, which are not ototoxic.

In patients ≥12 years of age, asthma is classified as intermittent if

In patients ≥12 years of age, asthma is classified as intermittent if symptoms are present ≤2 days per week, nighttime awakenings occur ≤2 times per month, an inhaler is required ≤2 days per week, and the FEV1 is >80% of predicted. Intermittent asthma does not interfere with normal activities and inhaled short-acting β2-agonists during symptomatic periods are usually sufficient treatment.

A 70-year-old male is being treated with medication for type 2 diabetes mellitus. Which one of the following hemoglobin A1c values is associated with the lowest mortality in this situation? (check one) A. 5.0%-5.9% B. 6.0%-6.9% C. 7.0%-7.9% D. 9.0%-9.9%

In patients ≥65 years of age treated with medication for type 2 diabetes mellitus, hemoglobin A1c values of 7%-8% have shown the greatest reduction in mortality in multiple studies. It is suggested that frequent hypoglycemia is associated with lower hemoglobin A1c values, and that presents a greater risk. Values over 9% are associated with greater mortality (SOR B). Thus, while the risk of complications increases linearly with hemoglobin A1c, mortality has a V-shaped curve.

This patient displays most of the criteria for borderline personality disorder. This is a maladaptive personality type that is present from a young age, with a strong genetic predisposition. It is estimated to be present in 1% of the general population and involves equal numbers of men and women; women seek care more often, however, leading to a disproportionate number of women being identified by medical providers. Which one of the following would be most beneficial for this patient? (check one) A. Clonazepam (Klonopin) B. Fluoxetine (Prozac) C. Quetiapine (Seroquel) D. Inpatient psychiatric admission E. Psychotherapy

Inpatient hospitalization may be an appropriate treatment option if the person is experiencing extreme difficulties in living and daily functioning, and pharmacotherapy may offer a mild degree of symptom relief. While these modalities have a role in certain patients, psychotherapy is considered the mainstay of therapy, especially in a relatively stable patient such as the one described.

A 62-year-old male sees you the day after returning from a 4-day cruise. He says he developed a fever and a productive cough on the day before the ship returned to Los Angeles following a trip down the coast of Baja California. He tells you that several other passengers had similar symptoms. The examination is remarkable for tachypnea and you hear crackles in both lungs.This patient's history should raise concerns about infection with which one of the following pathogens? (check one) A. Asian avian influenza A virus B. Coxiella burnetii C. Hantavirus D. Histoplasma capsulatum E. Leginella species

Legionella should be considered as a pathogen for community-acquired pneumonia when the patient has a history of a hotel stay or cruise ship travel within the past couple of weeks. Travel to or residence in Southeast Asia or East Asia is a risk factor for avian influenza, exposure to farm animals or parturient cats is a risk factor for Coxiella burnetii infection, exposure to bird or bat droppings is a risk factor for Histoplasma capsulatum infection, and travel to or residence in desert Southwest states with deer mouse exposure is a risk factor for Hantavirus infection.

Which one of the following is NOT considered a first-line treatment for head lice? (check one) A. Lindane 1% B. Malathion 0.5% (Ovide) C. Permethrin 1% (Nix) D. Pyrethrins 0.33%/pipernyl butoxide 4% (RID)

Lindane's efficacy has waned over the years and it is inconsistently ovicidal. Because of its neurotoxicity, lindane carries a black box warning and is specifically recommended only as second-line treatment by the FDA. Pyrethroid resistance is widespread, but permethrin is still considered to be a first-line treatment because of its favorable safety profile. The efficacy of malathion is attributed to its triple action with isopropyl alcohol and terpineol, likely making this a resistance-breaking formulation. The probability of simultaneously developing resistance to all three substances is small. Malathion is both ovicidal and pediculicidal.

A 20-year-old female visits your office for advice regarding contraceptives. She expresses interest in long-acting reversible contraception (LARC).Which one of the following is the recommended timing for LARC placement? (check one) A. Any time during the menstrual cycle B. 14 days before the anticipated onset of menses C. 7 days before the anticipated onset of menses D. 7 days after the onset of menses E. 14 days after the onset of menses

Long-acting reversible contraception (LARC) includes the copper IUD, levonorgestrel IUDs, and subdermal implants. LARCs can be placed at any point in the patient's menstrual cycle (SOR A). There should be evidence that the patient is not pregnant prior to placement.

A 76-year-old female presents with a 3-month history of watery diarrhea with up to 12episodes per day. She has no hematochezia and no travel history. You suspect microscopiccolitis.Which one of the following is the test of choice to confirm the diagnosis? (check one) A. A barium enema B. A stool test for calprotectin C. A celiac panel D. A biopsy of the colon E. A jejunal biopsy

Microscopic colitis is characterized by intermittent secretory diarrhea in older patients,although all ages can be affected. The cause is unknown, but there is some evidence that more than 6 months of NSAID use increases the risk. Only a biopsy from the transversecolon can confirm the diagnosis. Two histologic patterns are found: lymphocytic colitis andcollagenous colitis. The other diagnostic studies listed do not confirm the diagnosis.

What is mild persistent asthma?

Mild persistent asthma is defined as symptoms present >2 days per week but not daily, nighttime awakenings 3-4 times per month, and inhaler use >2 days per week but not daily and not more than once on any day. The FEV1 is >80% of predicted. Mild persistent asthma can cause minor limitations during normal activities and should be treated with low-dose inhaled corticosteroids (ICs).

A 2-year-old female is brought to your office because of a round lesion on her lip that appeared 2 days ago. Her temperature and all vital signs are normal. She has no past medical history and takes no medications. Further history reveals that she was playing with a toy trumpet in a busy store a few days before the lesion appeared. A physical examination reveals a 1-cm round lesion with crusting, and no other skin abnormalities.Which one of the following would be the best treatment at this time? (check one) A. Bacitracin B. Mupirocin (Bactroban) C. Neomycin D. Cephalexin (Keflex) E. Clindamycin (Cleocin)

Mupirocin (Bactroban) This patient has physical findings and a history consistent with impetigo, a skin infection caused by Staphylococcus aureus and/or Streptococcus pyogenes . Since she has only one lesion, systemic antibiotics are not required as they would be for a patient with extensive disease or multiple lesions. Although bacitracin and neomycin are commonly used, they are much less effective for impetigo than mupirocin, despite some reports of resistance to mupirocin (level A-1 evidence).

When is oxygen indicated in a child with viral bronchiolitis

No pharmacologic treatment shortens the course of viral bronchiolitis in a young child. Supplemental oxygen is indicated if the oxygen saturation falls below 90%, but otherwise the most effective treatment is simply supportive care (fluids, antipyretics, nasal bulb suction, etc.). None of the pharmacologic options listed are recommended in the treatment of bronchiolitis in this scenario.

You recently diagnosed diabetes mellitus in a 49-year-old male who also has chronic kidney disease and New York Heart Association class III heart failure. Laboratory studies are remarkable for a serum creatinine level of 2.0 mg/dL (N 0.6-1.2) and an estimated glomerular filtration rate of 40 mL/min/1.73 m2.Which one of the following classes of agents would be most appropriate for this patient? (check one) A. A biguanide such as metformin (Glucophage) B. A GLP-1 agonist such as liraglutide (Victoza) C. An SGLT2 inhibitor such as canagliflozin (Invokana) D. A thiazolidinedione such as rosiglitazone (Avandia)

Of the options given, only a GLP-1 agonist such as liraglutide could be used for this patient because of his comorbidities of chronic kidney disease and heart failure. Metformin is contraindicated in males with a creatinine level >1.5 mg/dL and in females with a creatinine level >1.4 mg/dL. SGLT2 inhibitors are not as safe or effective if the patient's estimated glomerular filtration rate (eGFR) is <50 mL/min/1.73 m2, and it is not recommended in patients with an eGFR <30 mL/min/1.73 m2. The initiation of rosiglitazone is contraindicated in patients with established New York Heart Association class III or class IV heart failure.

What is Osgood-Schlatter disease?

Osgood-Schlatter disease is seen in skeletally immature patients. Rapid growth of the femur can cause tight musculature in the quadriceps across the knee joint. It typically appears between the ages of 10 and 15, during periods of rapid growth. Pain and tenderness over the tibial tubercle and the distal patellar tendon is the most common presentation. The pain is aggravated by sports participation, but also occurs with normal daily activities and even at rest.

A patient returns to your office for a refill of oxycodone (Roxicodone), which he has been taking for 6 months for pain secondary to chronic osteomyelitis of his knee. His pain relief is adequate at a dosage of 10 mg every 6-8 hours. He took his prescribed dose approximately 4 hours prior to his visit. A urine drug screen using an enzyme-linked test is negative.Which one of the following would be most appropriate at this point? (check one) A. Stop prescribing pain medication for this patient B. Order chromatography C. Switch the patient to a codeine product and retest him D. Refer the patient to a pain management specialist

Oxycodone often is not detected by an immunoassay test and unexpected results require follow-up with a more accurate test such as gas chromatography/mass spectrometry or high-performance liquid chromatography. Codeine can be detected more accurately, but substituting codeine for oxycodone would be inappropriate. Pain management is reasonable if the patient is requiring large amounts of opiates, has failed treatment, or has a history of drug abuse. A pain management agreement should be initiated at the beginning of treatment with an opiate.

A 21-year-old female asks you about Papanicolaou (Pap) testing recommendations. You determine she is at average risk for cervical cancer and recommend which one of the following? (check one) A. Pap testing without HPV co-testing now and in 1 year if results are normal B. Pap testing without HPV co-testing now and in 3 years if results are normal C. Pap testing with HPV co-testing now and in 3 years if results are normal D. Pap testing with HPV co-testing now and in 5 years if results are normal E. HPV testing only, now and in 5 years if results are normal

Pap testing without HPV co-testing now and in 3 years if results are normal HPV testing is not recommended for screening in average-risk women younger than 30 years old. Cytology without HPV testing is recommended for screening every 3 years for an average-risk 21-year-old female.

A 9-year-old female presents with a 4-week history of right knee pain with activity. There is no history of trauma or recent illness. Your examination reveals lateral patellar tracking with extension of the knee.Which one of the following is the most likely diagnosis? (check one) A. Patellofemoral pain syndrome B. Osgood-Schlatter disease C. Growing pains D. Patellar tendinopathy E. Sever's disease

Patellofemoral pain syndrome is one of the most common causes of knee pain in children, particularly adolescent girls. Pain beneath the patella is the most common symptom. Squatting, running, and other vigorous activities exacerbate the pain. Walking up and down stairs is a classic cause of the pain, and pain with sitting for an extended period is also common. The physical examination reveals isolated tenderness with palpation at the medial and lateral aspects of the knee, and the grind test is also positive.

A 65-year-old female comes to your office in June for her Welcome to Medicare visit. She has no health concerns but believes she is due for some immunizations. She received Tdap 10 years ago. She has never had MMR and she did not receive influenza vaccine this past winter.According to the CDC adult immunization schedule, which one of the following is indicated for this patient? (check one) A. Influenza vaccine B. MMR C. 13-valent pneumococcal conjugate vaccine (PCV13, Prevnar 13) D. 23-valent pneumococcal polysaccharide vaccine (PPSV23, Pneumovax 23) E. Tdap

Patients 65 years of age and older should be vaccinated against Pneumococcus . The recommended sequence for a previously unvaccinated person is to give the 13-valent pneumococcal conjugate vaccine (PCV13) first, followed by the 23-valent pneumococcal polysaccharide vaccine (PPSV23) in 12 months. The interval was increased from 6 months in 2015. This patient has already received a Tdap booster so she requires only a Td booster. She does not need an MMR booster, as anyone born prior to 1957 is considered immune. Influenza vaccine should be delayed until fall. She is also due for herpes zoster vaccine if that was not offered when she turned 60.

Which one of the following is a cause of pseudohyponatremia? (check one) A. Hypercalcemia B. Hyperkalemia C. Hypermagnesemia D. Hyperphosphatemia E. Hypertriglyceridemia

Plasma sodium concentration measurements can be unreliable in patients with severe hyperlipidemia or hyperproteinemia (pseudohyponatremia). The other electrolyte abnormalities do not cause hyponatremia.

A 20-year-old male college student comes to your office to ask what he can do to prevent meningitis. His roommate was just hospitalized with invasive meningococcal disease. The patient has no symptoms at this time, a physical examination is normal, and he received meningococcal vaccine at the recommended times. He lives in a campus dormitory that houses 22 students.Which one of the following would be most appropriate? (check one) A. Reassure him that as long as he did not have direct contact with respiratory secretions he is at low risk of infection and does not need prophylaxis B. Reassure him that because of his immunization status he is at low risk of infection and does not need prophylaxis C. Tell him there is no preventive treatment so he should monitor his symptoms very closely and return immediately if he develops a fever, headache, or stiff neck D. Treat him and the rest of the students in the dormitory with a single dose of ciprofloxacin (Cipro)

Postexposure prophylaxis after exposure to invasive meningococcal disease is indicated for any close contact regardless of immunization status. Close contacts include those exposed in households, dormitories, or day care centers, and those who have direct contact with oral secretions. There are several options for prophylaxis, including ciprofloxacin, 500 mg orally one time; azithromycin, 500 mg orally one time; ceftriaxone, 250 mg intramuscularly one time; or rifampin, 600 mg orally twice daily for 2 days. Treatment should begin as soon as possible after exposure but no later than 14 days. While rifampin can be used for postexposure prophylaxis, a one-time dose is not adequate. This patient is at risk of infection due to his close contact with the source patient. Postexposure prophylaxis is indicated even for vaccinated patients.

A 36-year-old female calls your office because of a 2-day history of dysuria, urinary urgency, and urinary frequency. She has not had any fever, nausea, or vaginal discharge. She tells you her symptoms are similar to a previous urinary tract infection. She uses subdermal etonogestrel (Nexplanon) for contraception, takes no oral medications, and has no drug allergies.Which one of the following would be most appropriate at this point? (check one) A. Prescribe ciprofloxacin (Cipro) B. Prescribe nitrofurantoin (Macrobid, Macrodantin) C. Prescribe a urinary analgesic such as phenazopyridine (Pyridium) D. Ask the patient to come in today for evaluation E. Ask the patient to submit a urine specimen before you prescribe antibiotics

Prescribe nitrofurantoin (Macrobid, Macrodantin) Nonpregnant female patients who have typical UTI symptoms without signs of pyelonephritis (i.e., fever and nausea) or vaginitis can be treated safely and effectively by phone. Urine culture testing is not indicated for uncomplicated UTIs. It has been found that the traditional criterion for infection (100,000 colony-forming units/mL) is not sensitive for women with a UTI. Urine testing should be reserved for patients suspected of having pyelonephritis. There are three first-line antibiotics for uncomplicated UTI. These include nitrofurantoin for 5 days, trimethoprim/sulfamethoxazole for 3 days, and fosfomycin as a single dose (SOR A). Fluoroquinolones are second-line agents and are best reserved for more serious infections such as pyelonephritis.

A 63-year-old male presents to your office because his COPD is worsening. He indicates that his exercise tolerance is steadily decreasing but he can still walk approximately 100 yards on flat ground. His medications include formoterol (Perforomist) twice daily and albuterol (Proventil, Ventolin) as needed, which he requires only occasionally. He had an exacerbation requiring prednisone 7 months ago. He quit smoking 6 years ago. His oxygen saturation is 93% on room air and 89% after walking briskly for 8 minutes. His FEV1 is 1.91 L (62% of predicted) and his FEV1/FVC ratio is 0.57.Which one of the following is most likely to improve his progressive dyspnea? (check one) A. Stop formoterol and start fluticasone/salmeterol (Advair) B. Stop formoterol and start tiotropium C. Add tiotropium D. Start oxygen at 2 L/min with exertion E. Refer for pulmonary rehabilitation

Pulmonary rehabilitation has multidimensional benefits in COPD management, including improved exercise tolerance, quality of life, and mood. Aerobic exercise improves muscle mass, with high-intensity exercise proving more efficacious than low-intensity exercise. According to Global Initiative for Obstructive Lung Disease (GOLD) guidelines, an inhaled corticosteroid should be added for frequent exacerbations and an FEV1 <50%. Although it is possible that a combined long-acting β-agonist (LABA)/corticosteroid would improve dyspnea more than a LABA alone, the benefit would not likely be more than a pulmonary rehabilitation program.

What is Sever's disease

Sever's disease is an overuse syndrome most often seen between the ages of 9 and 14, and is related to osteochondrosis at the insertion of the Achilles tendon on the calcaneal tuberosity. It occurs during periods of rapid growth, causes heel pain during and after activity, and is relieved with rest. It is often related to beginning a new sport or the start of a season.

What is severe asthma?

Severe asthma is defined as symptoms present throughout the day, nighttime awakenings up to 7 times per week, inhaler use several times per day, and an FEV1 <60% of predicted. Normal activities are extremely limited by severe asthma. Treatment includes medium- to high-dose ICs with a LABA, and consultation with an asthma specialist is recommended. Omalizumab is also indicated for patients who have allergies.

A 70-year-old male presents with lower extremity pain. Increased pain with which one of the following would be most consistent with lumbar spinal stenosis? (check one) A. Lumbar spine extension B. Lumbar spine flexion C. Internal hip rotation D. Pressure against the lateral hip and trochanter

Spinal extension that increases lumbar lordosis decreases the cross-sectional area of the spinal canal, thereby compressing the spinal cord further. Walking downhill can cause this. Spinal flexion that decreases lordosis has the opposite effect and will usually improve the pain, as will sitting. Pain with internal hip rotation is characteristic of hip arthritis and is often felt in the groin. Pain in the lateral hip is more typical of trochanteric bursitis.

What are the guidelines for screening adults for diabetes

The American Diabetes Association recommends screening for all asymptomatic adults with a BMI >25.0 kg/m2 who have one or more additional risk factors for diabetes mellitus, and screening for all adults with no risk factors every 3 years beginning at age 45.

A rural community college has requested your guidance in offering a preventive health program to its students. The most appropriate plan would include which one of the following? (check one) A. Mammograms for female students B. Lead poisoning screening for all students C. Stool occult blood kits for all students D. Smoking cessation programs for students who smoke E. An annual routine physical examination for all students

The U.S. Preventive Services Task Force does not recommend routine physical examinations but recommends blood pressure screenings every 3-5 years for adults 18-39 years of age who are at low risk for hypertension. Mammograms are not recommended until age 40. Lead screening is recommended for at-risk individuals between 6 months and 6 years of age. Colorectal cancer screening for average-risk individuals is recommended at age 50. Counseling on tobacco use and other substance abuse is recommended as part of all routine preventive care.

Which one of the following is a frequent cause of cross-reactive food-allergy symptoms in latex-allergic individuals? (check one) A. Avocadoes B. Goat's milk C. Pecans D. Pastrami E. Peppermint

The majority of patients who are latex-allergic are believed to develop IgE antibodies that cross-react with some proteins in plant-derived foods. These food antigens do not survive the digestive process, and thus lack the capacity to sensitize after oral ingestion in the traditional food-allergy pathway. Antigenic similarity with proteins present in latex, to which an individual has already been sensitized, results in an indirect allergic response limited to the exposure that occurs prior to alteration by digestion, localized primarily in and around the oral cavity. The frequent association with certain fruits has been labeled the "latex-fruit syndrome." Although many fruits and vegetables have been implicated, fruits most commonly linked to this problem are bananas, avocadoes, and kiwi.

A 4-year-old male is brought to your office by his parents who are concerned that he isincreasingly "knock-kneed." His uncle required leg braces as a child, and the parents areworried about long-term gait abnormalities. On examination, the patient's knees touch when hestands and there is a 15° valgus angle at the knee. He walks with a stable gait.Which one of the following should you do now? (check one) A. Refer to orthopedics for therapeutic osteotomy B. Refer to physical therapy for customized bracing C. Prescribe quadriceps-strengthening exercises D. Provide reassurance to the patient and his family

This case is consistent with physiologic genu valgus, and the parents should be reassured. Toddlers under2 years of age typically have a varus angle at the knee (bowlegs). This transitions to physiologic genuvalgus, which gradually normalizes by around 6 years of age. As this condition is physiologic, therapies such as surgical intervention, special bracing, and exercise programs are not indicated.

A 62-year-old female with type 2 diabetes mellitus routinely has fasting blood glucose levels inthe 80-100 mg/dL range and her hemoglobin A1c level is 7.8%. She has been diligentlymonitoring her blood glucose levels and all are acceptable with the exception of elevated bedtime readings. She currently is on insulin glargine (Lantus), 18 U at night.Which one of the following changes would be most appropriate for this patient? (check one) A. Adding rapid-acting insulin at breakfast B. Adding rapid-acting insulin at lunch C. Adding rapid-acting insulin at dinner D. Increasing the nightly insulin glargine dose E. Increasing the insulin glargine dosage and giving two-thirds in the morning and one-third at night

This patient continues to have an elevated hemoglobin A1c and bedtime hyperglycemia. The addition of arapid-acting insulin at dinner would be the next step in management. For patients exhibiting blood glucoseelevations before dinner, the addition of rapid-acting insulin at lunch is preferred. For patients withelevations before lunch, rapid-acting insulin with breakfast would most likely improve glucose control.Increasing or splitting the insulin glargine would be unlikely to improve management.

A radiograph reveals a proximal fifth metatarsal metadiaphyseal fracture. The patient is eager to return to competitive dancing as soon as possible.Which one of the following would be the best treatment for this patient? (check one) A. Full weight bearing with the use of a compression dressing as needed for pain and swelling B. A posterior splint with no weight bearing for 4 weeks C. A walking cast for 6 weeks D. Surgical referral

This patient has a Jones fracture. The treatment plan for this type of fracture needs to account for the activity level of the patient. It has been shown that active patients have shorter healing times and return to activity sooner with surgical management. A competitive dancer would be best managed with surgery. If the nonsurgical option is chosen the patient is given an initial posterior splint and followed up in 3-5 days, then placed in a short non-weight-bearing cast for 6 weeks, at which time a repeat radiograph is taken. If the radiograph shows healing, the patient can return to gradual weight bearing. If the radiograph does not show proper healing, then the period of non-weight bearing is extended.

A 60-year-old male returns for a reevaluation of his asthma that you have been working to control. In the past he has been adequately maintained with daily use of inhaled fluticasone/salmeterol (Advair), along with montelukast (Singulair). Last month he experienced an exacerbation of his asthma that responded well to oral prednisone. However, each time you have attempted to wean him off the prednisone his asthma symptoms have returned.Which one of the following would be most appropriate at this time? (check one) A. Adding oral azithromycin (Zithromax) B. Adding oral methotrexate C. Adding oral theophylline D. Adding long-term oral prednisone E. Increasing the dosage of the corticosteroid

This patient has severe asthma that is not responding to a moderate dose of an inhaled corticosteroid, a leukotriene inhibitor, and a long-acting β-agonist. The next appropriate step is to add a stronger dose of inhaled corticosteroid. Methotrexate and azithromycin are considered inappropriate therapies. Theophylline and low-dose oral prednisone are considered appropriate steps if the patient does not respond to high doses of an inhaled corticosteroid. Other reasonable options for the treatment of severe asthma would be a muscarinic antagonist such as tiotropium, or assessing for the presence of IgE-dependent allergic asthma that may respond to omalizumab.

A 66-year-old asymptomatic male presents with a 6-month history of ongoing pruritus. A physical examination is normal. Laboratory studies are also normal except for an LDL-cholesterol level of 150 mg/dL, a free T4 level of 1.4 ng/dL (N 0.9-2.3), and a TSH level of 6.22 μU/mL (N 0.4-4.5).Which one of the following would be most appropriate at this point? (check one) A. No treatment at this time and a repeat TSH level in 3 months B. Testing for thyroid antibodies and treatment with levothyroxine if the test is negative C. A 131I uptake and scan with no treatment pending results D. Liothyronine (Cytomel) E. Levothyroxine (Synthroid)

This patient has subclinical hypothyroidism and should have a TSH level repeated in 1-3 months, as TSH may fluctuate in patients without thyroid disease and return to normal on subsequent testing. In a patient with a normal free T4 the TSH level must be >10 μU/mL for a diagnosis of hypothyroidism. Mild TSH elevations may be a normal manifestation of aging.

A 2-year-old male has a 3-day history of a runny nose and cough, and a 2-day history of fever reaching 40.0°C (104.0°F). He woke up with a rash this morning. His appetite is good and his activity level is normal. On examination the child is afebrile with normal vital signs, and has a fine, maculopapular, erythematous rash on the trunk and extremities. The remainder of the examination is normal.Which one of the following is the most likely cause of this patient's rash? (check one) A. Atopic dermatitis B. Erythema infectiosum C. Molluscum contagiosum D. Pityriasis rosea E. Roseola infantum

This patient has the classic presentation for roseola infantum, which is caused by human herpesvirus 6. The typical history includes a high fever in a child with either mild upper respiratory symptoms or no other symptoms. After the fever subsides, a rash will appear. The rash is self-limited and no treatment is required.

A 45-year-old male sees you for a hypertension follow-up visit and points out a nonpainful bump that he noted on his neck 3-4 months ago. He does not use tobacco products and is otherwise healthy. He has not had any fever or pain. His blood pressure is controlled with lisinopril (Prinivil, Zestril). On further evaluation a firm, mobile, nontender 1.5-cm right anterior cervical lymph node is noted.Which one of the following would be most appropriate at this point? (check one) A. Watchful waiting B. Discontinuation of lisinopril C. Antibiotics D. Corticosteroids E. Fine-needle aspiration

This patient is at high risk for malignancy based on his age, sex, and lymphadenopathy for more than 4-6 weeks. Fine-needle aspiration is an acceptable first-line test to evaluate for a reactive node versus malignancy. Further testing may be necessary to confirm the diagnosis.

A 60-year-old male with COPD returns for a follow-up visit. He has been treated twice for pneumonia in the past year and is using his albuterol metered-dose inhaler (Proventil, Ventolin) several times a day. His FEV1 is 60% of predicted.To reduce symptoms and prevent hospitalization, which one of the following would be the most appropriate medication to add to this patient's treatment regimen at this time? (check one) A. Roflumilast (Daliresp) B. A long-acting β2-agonist bronchodilator C. A long-acting β2-agonist and an inhaled corticosteroid D. A long-acting anticholinergic inhaler E. Long-acting theophylline

To decrease the patient's frequent use of a short-acting bronchodilator, the addition of a long-acting anticholinergic inhaler for maintenance is most reasonable. Such medications have been demonstrated to improve quality of life and reduce hospitalizations. Evidence has demonstrated that a β-agonist combined with an inhaled corticosteroid would not be as preferable in this case, as they can increase the incidence of pneumonia. Long-acting theophylline and roflumilast may be added to the regimen of patients still symptomatic on triple therapy with long-acting β2-agonists, inhaled corticosteroids, and anticholinergic bronchodilators.

A 60-year-old female sees you because she has recently lost 20 lb without trying and is having trouble swallowing. Her other medical problems include obesity, tobacco abuse, and GERD.Which one of the following is the most appropriate diagnostic test in this situation? (check one) A. A barium swallow B. Endoscopic esophageal ultrasonography C. Chest CT D. Upper endoscopy

Upper endoscopy Dysphagia alone or with unintentional weight loss is the most common presenting symptom of esophageal cancer. Adenocarcinoma is the most common esophageal cancer in developed nations, and risk factors include GERD, obesity, and tobacco abuse. Upper endoscopy is the recommended diagnostic tool (SOR B). If cancer is confirmed, CT and PET scanning are useful for staging.

What is moderate persistent asthma?

With moderate persistent asthma, symptoms are present daily, nighttime awakenings occur >1 time per week but not nightly, and an inhaler is required daily. A patient with moderate persistent asthma has an FEV1 that is 60%-80% of predicted and can experience some limitations during normal activities. Moderate persistent asthma is treated with a combination of low-dose ICs and long-acting β2-agonists (LABA) or medium-dose ICs as monotherapy.

How often should blood pressure screenings be performed

blood pressure screenings every 3-5 years for adults 18-39 years of age who are at low risk for hypertension.

A pleural protein to serum protein ratio >0.5 or a pleural fluid LDH to serum LDH ratio >0.6 suggests an _____ effusion.

exudative Lower ratios suggest a transudative process.

What is anchoring bias?

occurs when decision makers are influenced by the first information received about a decision, even if it is irrelevant

Dupuytren's contracture is characterized by changes in the ___________, with progressive thickening and nodule formation that can progress to a contracture of the associated finger. The _____ finger is most commonly affected. Pitting or dimpling can occur over the nodule because of the connection with the skin.

palmar fascia fourth

Cirrhosis with ascites is a cause of _________ effusion. Pleural effusions associated with malignancy, pneumonia, viral illness, and asbestosis tend to be _______ effusions

transudative exudative.


Related study sets

Chapter 6: Microscopic Examination of Urine

View Set